If you're seeing this message, it means we're having trouble loading external resources on our website.

If you're behind a web filter, please make sure that the domains *.kastatic.org and *.kasandbox.org are unblocked.

Main content

Ordering setup questions | Video lesson

Watch one way to approach the questions on an ordering setup on the analytical reasoning section of the LSAT.

Want to join the conversation?

Video transcript

- [Instructor] Now we'll do the questions from the ordering setup about objects being stacked. Before you continue with this video, make sure that you've watched that previous video in which, we worked through the setup and all the deductions we were able to make from the rules. In this video, which we're going to keep under 10 minutes. I'm not gonna have time to explain why all of the wrong choices are wrong. Instead, I'm going to show you how to work through each problem to get to the answer quickly using the deductions we've made so that you can move on with confidence without having to slog through all of the wrong choices. That's really all you're going to have time for on test day anyway, so let's see how efficient we can be. Our first question asks, which one of the following could be the objects in the stack listed from bottom to top? This is an overview question and it's essentially asking us, which choice doesn't break any of the rules? The most efficient way to tackle these question types tends to be starting with the rules, going through them one by one, and knocking out the choices that violate each rule. One thing I will draw your attention to here is that the choices aren't listed vertically the way our diagram is. When they tell us that the objects are listed from bottom to top, it's probably a good idea to number right above the choices like this. That way you won't get confused and accidentally mix up which is the bottom and which is the top. Rule number one tells us that L is immediately above K. That means that we can eliminate C where L is below K. Rule number two tells us that J is not in position one. We'll look at the choices and see that in choice A, J is in position one so we can eliminate choice A. Rule three is that M can't be right below K. Because we label where the bottom and the top is in each choice, we can see that choice D has M being right below K. We can cross that one out as well. Finally, rule four tells us that N must be either position one or position five. Choice B, we see that N is in position four. We're left with choice E and that is our answer. We only ever have to go through the rules once for these overview questions and by eliminating, we can get to our answer. This questions asks, if J is on top of the stack, which one of the following must be true? This is a must be true question and it gives us a new condition, a temporary condition just for this question that J is on top. We can re-draw our initial diagram and incorporate the new rule that J is on top. We can just draw a bare bones diagram here since we still have our initial diagram on the page. If J is on top of the stack, what we wanna do is combine that with our original rules to see what we can deduce from that. If J is on top, which is in spot five, that means that N must be in spot one according to rule four. Now we've got J on top and on the bottom, we have to account for K, L, and M. What do we know about K, L, and M? We know that L is immediately above K based on the first rule but, we also know that M can't be immediately below K. That means that L must be in three, K is in two, so that M can be fourth and not be below K. Which one of the following choices must be true? As we scan through the choices and look at what must be true based on our new diagram for this question, the answer is E, M must be in position four. All of the other choices could be false and in fact, in this case they must be false. If L is in position two in the stack, which one of the follow could be true? This question asks us for what could be true, given a new condition that L is second. Again, let's re-draw a bare bones copy of our initial diagram and incorporate the new rule that L is in position two. What does this mean if L is in two? We know from the rules that L is immediately above K in the stack. If L is two, then K must be one. If K is one, that leaves N to be in position five based on rule four. We have to account for J and M still. Looking at the rules, it doesn't seem that we can figure out exactly where they go. Either J or M is in four, and either J or M is in three. We're asked for what could be true. Choice A says that J is in position three, that could be true. All of the other choices must be false based on our diagram for this question. This question asks, which one of the following could be in any one of the five positions in the stack? This is a could be true question and we aren't given a new condition. We don't need to re-draw anything. We simply have to use what we know of the initial setup. Each choice represents one of the objects. Let's just go through each choice and determine whether that object can be anywhere in the stack that it wants to be without breaking any rules. In other words, if there is an acceptable scenario in which that object could be one, two, three, four, or five. Choice A. We know that J can't be first because we already deduced that in our initial setup. We can eliminate this choice. B has object K and K can't be fifth. We've deduced that already so we can eliminate B. C has object L which we know can't be first so we can eliminate C. Choice D has object M. M's interesting because we do have a rule that shows that M is not immediately below K in this stack but, M can be anywhere in a certain situation. It's only restriction is that K can't be right above it which is fine. There are acceptable scenarios in which M could be one, two, three, four, or five. This is gonna be our answer but, quickly checking E we can see that N can't be second, third, or fourth. Our answer is D, M could be in any one of the five positions in the stack. If M is neither directly above nor directly below J in the stack, which one of the following objects must be in position four? I'm not gonna re-draw quite yet here because they're not telling us exactly where M or J goes. What they're telling us is basically two new rules. One which is that M can't be directly above J, and the other which is that M cannot be directly below J. This doesn't seem to make anything obviously true at the moment. We just know that M can't be next to J and we also know from the original rules that M can't be below K, but that doesn't tell us what M is next to or where M is. Let's not keep staring at it. On test day, if you feel stuck, look at what else you know to be true. We do know from the rules that N can only be first or fifth. That means that we can draw multiple scenarios, one in which N is fifth and another in which N is first and maybe by drawing out these scenarios, it'll make some things surface up more apparently. In scenario one, we have N in five. We know that L, K is a pair, but if we put L, K in certain spots, that would leave J, M as a pair and the question specifically tells us that J, M can't be a pair. For example, L, K can't be third and fourth because that would leave J and M next to each other. L and K also can't be first and second because we again have J and M next to each other. In this first scenario, L would have to be third and K would have to be second. That's the only way that we could break J and M up. But wait, which object goes on the bottom here? It can't be M because rule three tells us that M isn't right below K. It also can't be J because rule tells us that J can't be first. This shows us that this scenario doesn't actually work. We can therefore deduce that N can't be fifth. That means that N has to be first like it is in scenario two so we're making progress. Let's cross this first scenario out so that we don't accidentally use it any time later and we'll look at scenario two. Here in scenario two where N is first, we can move a little more quickly here because we know that one of the keys to this question is keeping J and M separated. Once again, the L, K pair can't be second and third and it also can't be fourth and fifth because we need to keep J and M separated. The only place the L, K pair can go is third and fourth. That means that J is immediately below K since M can't be and therefore M goes on top. This scenario doesn't break any rules and the question asks, which object must be in position four. Now we know that it's L and the answer is C.